4120 1400
NOTEPAD
Results
of 168 questions answered correctly

You have reached of 168 points, ( %)

Your time

Question 1 of 168

1. A 38-year-old male works within the range of ionizing radiation. At a routi- ne medical examination he presents no problems. In blood: RBCs - 4, 5·1012/l, Hb- 80 g/l, WBCs - 2, 8 · 109/l, thrombocytes - 30 · 109/l. Decide if this person can work with sources of ionizing radiation:

Explanation

2. A 50-year-old patient has worked at a chemical plant for 15 years. His work involved using xylene solvent. The patient was hospitalized with suspected chronic intoxication. He was found to have anemic syndrome. What is the first- priority measure of secondary anemia prevention?

Explanation

Prevention includes a wide range of activities — known as “interventions” — aimed at reducing risks or threats to health. Three categories of prevention: primary, secondary and tertiary.

Primary prevention aims to prevent disease or injury before it ever occurs. This is done by preventing exposures to hazards that cause disease or injury, altering unhealthy or unsafe behaviours that can lead to disease or injury, and increasing resistance to disease or injury should exposure occur. 

Secondary prevention aims to reduce the impact of a disease or injury that has already occurred. 

Tertiary prevention aims to soften the impact of an ongoing illness or injury that has lasting effects. 

In this case, the patient already has anemic syndrome with suspected chronic intoxication. Now, the aim is to prevent further exposure by changing his job. Primary prevention would have involved not working with the Xylene solvent in the first place.

 

All other options listed are geared towards treating the condition, however, the question is concerned about preventing further anemic complications which is to completely prevent exposure to the Xylene solvent.

3. Explosion of a tank with benzene at a chemical plant has killed and wounded a large number of people. There are over 50 victims with burns, mechanical traumas and intoxication. Specify the main elements of medical care and evacuation of population in this situation:

Explanation

In this case, A Triage system is to be followed. Triage is a procedure followed in a medical emergency. It involves the sorting of and allocation of treatment to patients and especially battle and disaster victims according to a system of priorities designed to maximize the number of survivors

These individuals should be allocated into categories such as: Minor, Deceased, Immediate and delayed. Those in the immediate category require instant medical assistance. Lastly, all 50 victims should be evacuated from the site of explosion. 

 
4. It is planned to build a multi- disciplinary hospital with 500 beds in a town. Specify the location of a polyclinic within the medical centre:

Explanation

5. Survey radiograph of a 52-year-old worker of an agglomeration plant (28 years of experience, the concentration of metal dust is 22-37 mg/m3) shows mildly pronounced interstitial fibrosis with di- ffused contrast well-defined small nodular shadows. The patient has no complaints. Pulmonary function is not compromised. What is the provisional diagnosis?

Explanation

Agglomeration plants are factories that work mainly with Iron ore materials together with other products, refining them into finished goods. This  patient has an accumulation of iron dust in his system as a result of long term exposure over the years. Pneumoconiosis is a restrictive lung disease  caused by the inhalation of dust leading to fibrosis. Depending on the type of dust inhaled, different types exist. They include; Siderosis (iron ore), byssinosis (cotton), anthracosis (coal), Asbestosis (asbestos), silicosis (silica dust).

6. A 45-year-old female patient has worked as a painter for 14 years. Contacts with synthetic paint result in face skin redness, swelling, intense itching, oozi- ng lesions. The symptoms disappear after the exposure to chemical agents, but recur even at the smell of paint. The symptom intensity progresses with relapses. Make a provisional diagnosis:

Explanation

Eczema (dermatitis ) is a chronic inflammatory disease of the skin characterized by the presence of red, itchy, dry scaly rashes. Occupational or professional  eczema (dermatitis) is gotten from contact with certain chemicals in the course of working (in this case due to contact with the chemicals in paint ). 

Allergic dermatitis is a form of contact dermatitis that results from immune reactions towards certain irritants. This term can be used if it’s unrelated to the patient’s work.

 
7. During the periodic medical examination an assembly fitter (works on soldering details) didn’t report any health problems. Closer examination revealed signs of asthenic-vegetative syndrome. Blood included red blood cells with basophilic aggregations and a somewhat higher number of reticulocytes, urine had a high concentration of delta-aminolevulinic acid. The complex of symptoms indicates the initial stage of chronic intoxication with:

Explanation

 

Delta-aminolevulinic acid dehydratase is an enzyme necessary for the synthesis of heme. In lead poisoning, this enzyme is inhibited leading to anemia and an increased delta aminolevulinic acid content in urine. N/B a high reticulocyte number means an increase in RBC production ( new cells) and is observed in cases such as bleeding, high altitude or anemia. The above patient presents with signs of asthenic vegetative syndrome coupled with the above stated reasons confirming a case of lead poisoning.

8. Examination of a group of persons living on the same territory revealed the following common symptoms: dark-yellow pigmentation of the tooth enamel, diffuse osteoporosis of bone apparatus, ossification of ligaments and joints, functional disorders of the central nervous system. This condition may be caused by the excessive concentration of the following microelement in food or drinking water:

Explanation

 

Presence of dark-yellow pigmentation of the tooth enamel, diffuse osteoporosis of bone apparatus, ossification of ligaments and joints are specific symptoms that indicate Fluorosis. This is a condition that occurs as a result of excessive intake of fluorine.

9. A patient is on the sick leave for 4 months continuously from the date of injury. The treatment is going to last for 1-2 months more. Who has the right to extend the duration of medical certificate for this patient?

Explanation

The right to extend a medical certificate (in this case a sick list) of invalidity alongside with the medical treatment by the doctor is granted by:

• Chief medical officer (assistant of the chief medical officer on expertise of ability for work);

• Manager of department;

• Medical-consulting commission also known as medical expertise committee

The medical-consulting commission (MCC) is formed in case the doctors staff is above 15 doctors.

 
10. A rural hospital serves 6200 people. Preventive examinations were planned for 560 farm workers with different risk factors. 400 workers underwent preventi- ve examination. 120 individuals were found to have cardiovascular problems, 90 of them were registered with a di- spensary department for health care mai- ntenance. Which indicator should be used to assess the organization of health care maintenance at the hospital?

Explanation

11. A family consists of 5 persons. The husband is a stope miner. His spouse is a housewife. Their 20-year-old daughter works as a kindergarten teacher. Their 18-year-old son is a student. The grandmother is a pensioner, she has diabetes. Which member of this family can be primarily classed among the group of persons with a high risk of tuberculosis in the planning of preventive examinations for tuberculosis?

Explanation

Generally, persons at high risk for developing TB disease fall into two categories:

  • Persons who have been recently infected with TB bacteria

  • Persons with medical conditions that weaken the immune system

Persons in the second group include individuals with the following diseases; HIV infection (the virus that causes AIDS), Substance abuse, Silicosis, Diabetes mellitus, Severe kidney disease, Low body weight, Organ transplants, Head and neck cancer, Medical treatments such as corticosteroids or organ transplant, Specialized treatment for rheumatoid arthritis or Crohn’s disease

12. During medical monitoring of labor and professional training of 6th grade schoolchildren the school doctor found that lessons in carpentry workshop are held on Friday as the fourth and fifth lesson of a total of 90 minutes, the motor density at the first lesson is 69%, and 65% at the second. Does the lesson organizati- on meet the hygienic requirements?

Explanation

13. During coal extraction in a mine the concentration of coal dust in the working area is 450 mg/m3 (MPC is 10 mg/m3). What occupational respiratory disease may develop in miners?

Explanation

 

Pneumoconiosis is a restrictive lung disease  caused by the inhalation of dust leading to fibrosis. Depending on the type of dust inhaled, different types exist. They include; Siderosis (iron ore), byssinosis (cotton), anthracosis (coal), Asbestosis (asbestos), silicosis (silica dust).the maximum permissible concentration of coal is 10 mg/m3, this patient is in an area where the coal content is  450 mg/m3. Accumulation of coal dust in the lungs will lead to anthracosis. 

14. The share of circulatory diseases was 15% in the total of registered diseases among city population. What statistic indicator is it?

Explanation

The extensive statistical indicator is used to determine a structure of a disease ( in this case, how many percent of the total diseased fall under cardiovascular cases) i.e., it shows, what part from the general number of all diseases is made with this or that disease which enters into total.

The intensive parameter characterizes frequency or distribution. It shows how frequently the given phenomenon occurs in the given environment.

 
15. During a regular medical examination at a metallurgical plant 20% of workers were found overweight (body weight was 5-14% higher than normal), and had early signs of obesity (grade I-II) with Quetelet index from 26 to 30. What products share must be reduced in the diet of this group of people in the first place in order to normalize their body weight?

Explanation

 

These factory workers are obesed, with BMI (quetelet index) - 26-30. In relation to diet, carbohydrate and fatty food groups play a large role in obesity; therefore in terms of diet food materials that contain large contents of these groups should be controlled. This includes bakery products such as flour, wheat etc.

16. Chief physician of a polyclinic encharged a district doctor with a task to determine the pathological prevalence of disease N in his district. What document allows to estimate the disease prevalence in the population of a medical district?

Explanation

17. In an urban settlement situated on the riverbank an outbreak of hepatitis A was registered. The disease might have water origin. This assumption can be confirmed by growth of the following indicators of water quality:

Explanation

 

Coliphages are microbial indicators specified in the Ground Water Rule (GWR) that can be used to monitor for potential fecal contamination of drinking water. Number of bacteria of group of an intestinal rod (coli-index) in 1000 ml of water - no more than 3.Common microbial number in one ml of water - no more than 100.

18. The institutions which take part in medical examinations can be prevention and treatment facilities, medical board of Ministry of Defense, medical board of Ministry of Home Affairs, medico-social expert commissions, forensic medical boards etc. What institutions are responsi- ble for temporary disability examination?

Explanation

19. A city somatic hospital with 300 beds consists of the main building which houses the therapeutic and surgical departments. Several separate buildings house the maternity, pediatric and radiologic departments that are connected to the main building by underground walkways and above-ground covered skybridges. Specify the building system of the hospital:

Explanation

20. Sanitary examination of the burns unit for adults revealed that 4-bed wards had an area of 28 m2. What is the minimally required ward area for this department?

Explanation

21. An employee was on a business trip to another city, where he fell ill and was hospitalized. The sick leave certificate can be issued:

Explanation

22. In order to reduce weed growth on agricultural land, some herbicides have been used for a long time. In terms of environmental stability these herbicides are rated as stable. Specify the most likely route of their entry into the human body:

Explanation

Herbicides are chemicals (a form of pesticides) used in killing unwanted plants; when applied, they come in contact with the soil and may affect food products (plants) which will cause harm to the consumer. (soil-plants-humans).
23. A selective population research study was aimed at exploring the effect of air emissions from a metallurgical plant on the obstructive bronchitis morbidity in a city. The calculated correlation coefficient was +0,79. Evaluate the strength and direction of the relationship:

Explanation

The Spearman's Rank Correlation Coefficient is used to discover the strength of a link between two sets of data. This example looks at the strength of the link between the effect of air emissions from a metallurgical plant on obstructive bronchitis morbidity.

Correlation is a bivariate analysis that measures the strength of association between two variables and the direction of the relationship.  In terms of the strength of relationship, the value of the correlation coefficient varies between +1 and -1.  A value of ± 1 indicates a perfect degree of association between the two variables.  As the correlation coefficient value goes towards 0, the relationship between the two variables will be weaker.  A correlation coefficient of zero indicates that no relationship exists between the variables. The direction of the relationship is indicated by the sign of the coefficient; a + sign indicates a positive/direct relationship and a – sign indicates a negative relationship.

An inverse correlation, also known as negative correlation, is a contrary relationship between two variables such that when the value of one variable is high then the value of the other variable is probably low.

The value of the correlation coefficient ‘r’ can range from 0.0, indicating no relationship between the two variables, to positive or negative 1.0, indicating a strong linear relationship between the two variables. 

Value of r

Indications

0.0

No linear relationship between the two variables

+1.0

Strong positive linear relationship; as X increases in value, Y increases in value also; or as X decreases in value, Y decreases also.

-1.0

Strong inverse linear relationship; as X increases in value, Y decreases in value; or as X decreases in value, Y increases in value.

 

Therefore, a value of +0.79 is very close to +1.0 and that will give us a strong positive/direct relationship.

24. A general practitioner visited a 2-year-old child and diagnosed him with measles. The child attends a nursery, has a 5-year-old sister. What document must be filled in for the effective antiepidemic measures in the given health locality?

Explanation

This Patient has a measles infection (an infectious disease).  A form that will notify the infectious disease department and provide the necessary information to help curtail the spread should be filled.
25. Hygienic expertise of a sample taken from the batch of grain revealed that 2% of grains were infected with microscopic Fusarium fungi. On the ground of laboratory analyses this batch of grain should be:

Explanation

0-7% – the seed may not need to be treated with a registered seed treatment.

8-14% – the seed should be treated with a registered seed treatment.

> 14% – the seed should not be planted.

 
26. Factory’s sectorial doctor chooses a group of chronically ill people. He takes into account the duration of etiologically related cases with a temporary disability over the last year in each of the workers. The employees will fall into this group if the duration is:

Explanation

27. 6 people live in a modern flat with the total area of 60 m2. There are TV-video equipment, radios, microwave ovens, computer. The residents of the flat complain of bad health, occasional headaches, arrhythmia, conjunctivitis. What is the most likely cause of this condition?

Explanation

 

Many household appliances produce electromagnetic fields for example, television, microwave oven, radios etc. The effects of electromagnetic waves on health are vast. Immune, neurological, endocrine systems are majorly affected.

28. A man abused alcohol, drank away the property and wages thus getting hi- mself, his wife and two underage children into deep financial problems. He was regi- stered in a local drug abuse clinic. His wife asked a family doctor, what kind of petiti- on she could file in court:

Explanation

29. Analysis of organization of medi- cal care in a regional centre has shown that every year about 12% of patients receive inpatient care for diseases that don’t require round-the-clock monitori- ng and intensive care. What are the most appropriate organizational changes requi- red to address this problem?

Explanation

30. Doctors of a polyclinic conduct a statistical research of the disease outcomes in two groups of patients (those registered with dispensary departments and unregistered ones), depending on age and level of hygiene. What type of statistical tables would be most suitable for profound analysis of the interrelation between the above-mentioned variables?

Explanation

31. Head of a department and a trade-union group have appealed to the head of a hospital about dismissal of the senior nurse who has 17-year record of service. The facts of charge were confirmed and recognized by the nurse herself. The nurse lives with a daughter (who is divorced and unemployed) and a 9-month-old grandson. Make an administrative decision:

Explanation

32. Within a year, in a maternity hospital there were 616 livebirths, one stillbirth, one infant died on the 5th day of life. What index can most accurately describe this situation?

Explanation

Perinatal mortality refers to the number of stillbirths and deaths in the first week of life (early neonatal mortality).

The infant mortality rate is defined as the number of deaths of children under one year of age.

 
33. In one of the surgical departments the quality assurance testing of sterilization of surgical instruments was performed. After an instrument had been treated with 1% phenolphthalein, the solution turned pink. This indicates that the instrument has:

Explanation

34. Study of natural illumination for a workplace in a secondary school classroom revealed that the angle of sunlight incidence was 25*, window opening angle - 3*, window-to-floor area ratio - 1:4, daylight ratio - 0,5%, depth ratio - 2. What indicators do not meet hygienic standards?
35. Bakers at bread production work in conditions of high temperature and high heat radiation. What is used to increase the body’s resistance to the unfavorable effects of these harmful work environment factors?

Explanation

 

Vitamin preparations such as vitamin D helps against radiation; Vitamin E acts as an antioxidant which destroys free radicals (reactive oxygen species).

36. A study of the structure of death causes in the urban population revealed that cardiovascular diseases accounted for 55,0% of all deaths. What statistic value represents these data?

Explanation

The extensive statistical indicator is used to determine a structure of a disease ( in this case, how many percent ( the proportion) of the total diseased fall under cardiovascular cases) i.e., it shows, what part from the general number of all diseases is made with this or that disease which enters into total.

The intensive parameter characterizes frequency or distribution. It shows how frequently the given phenomenon occurs in the given environment. Examples of Intensive parameters include morbidity rate, mortality rate, lethality rate etc. 

 
37. A 51-year-old female is a weaving factory worker with 15 years of service record. During a regular preventive examination she complained of frequent headaches, poor sleep, tingling in the heart, irritability, rapid fatigability, hearing impairment. For years, the noise level has exceeded the maximum allowable concentration by 10-15 dB. A year ago, the patient underwent a course of treatment for essential hypertension. Specify the most likely diagnosis:

Explanation

From anamnesis, it is stated that the patient has worked for a very long time in an environment in which the noise level exceeds the maximum allowable concentration. The complaints presented on preventive examination are as a result of prolonged exposure to noise over a long period of time. 

Asthenovegetative syndrome: weakness, severe fatigue, poor performance and mood, anxiety, depressive mood, hypochondria.

Neurasthenia: a condition that is characterized especially by physical and mental exhaustion usually with accompanying symptoms (such as headache and irritability), is of unknown cause but is often associated with depression or emotional stress, and is sometimes considered similar to or identical with chronic fatigue syndrome.

 
38. Some of the population of a city distri- ct have uneven teeth color. The individuals have white spots, transverse brown stripes on the incisors. Occurrence of these symptoms is associated with the quality of drinking water from a deep well. Which of the following components of water can be the cause of the disease?

Explanation

These individuals are experiencing Dental fluorosis which is a condition caused by excess fluorine in drinking water leading to inadequate enamel formation. Key signs include brown and white stripes on the teeth. Water fluoridation should be carried out in order to correct the amount of fluorine.
39. A 48-year-old male in-patient undergoes treatment for essential hypertension of II- B stage. It is known from history that he works in a design engineering office. His job involves neuro-emotional stress. Which of these foodstuffs do not stimulate the central nervous system and can be recommended for the patient?

Explanation

40. Two years ago, a 46-year-old patient was diagnosed with stage I silicosis. Currently the patient complains of escalating dyspnea, pain in the infrascapular regions. Radiograph shows a diffuse enhancement and distorti- on of lung markings, as well as multiple nodular shadows 2-4 mm in diameter. There is interlobar pleural density on the right. Dense shadows are found in the hilar regions. Specify the form of radiographic pulmonary fibrosis in this case:

Explanation

41. During dressing of a poorly-granulating wound Pseudomonas aeruginosa infection was revealed. What medication would be optimal for the wound d-bridement?

Explanation

Boric acid Medical uses include; 

 
  • antiseptic in burns, cuts and dressing.

  • Vaginal douche in bacterial vaginosis and candidiasis.

  • Prevention of athlete’s foot, otitis externa.

42. Examination of a dead man who died from hanging revealed that cadaver spots disappeared when pressed upon and restored after 50 seconds, rigor mortis was moderately expressed only in the masticatory muscles and the muscles of neck and fingers. Body temperature was 31*C. Specify the time of death:

Explanation

Forensic pathologists are essentially required to estimate the time since death (TSD) to assist in death investigation. The early post-mortem phase is most frequently estimated using the classical triad of post-mortem changes – rigor mortis, livor mortis, and algor mortis.

Rigor mortis is the post-mortem stiffening of muscles, caused by the depletion of adenosine triphosphate (ATP) from the muscles, which is necessary for the breakdown of actin-myosin filaments in the muscle fibers.

Fully developed rigor mortis is an easily identifiable and reliable indicator that death has occurred. The time of onset is variable but it is usually considered to appear between 1 and 6 hours after death. Smaller muscles over the face – around the eyes, around the mouth (masticatory muscles), etc. are the muscles where rigor mortis first appears, followed by rigor mortis of the muscles in the hands and upper limbs and finally appears in the large muscles of the lower limbs. Rigor mortis appears approximately 2 hours after death in the muscles of the face, progresses to the limbs over the next few hours, completing between 6 to 8 hours after death.

 
43. An employee of a petrol station with 15 years of service record having contact with ethylated gasoline presents with memory impairment, bradycardia, sensation of having a hair in the mouth, skin paresthesia. In this case, one can assume intoxication with the following substance:

Explanation

From anamnesis, this patient has been exposed to Ethylated gasoline for the nature of his Job. Tetraethyl Lead is a lead compound found in Ethylated gasoline. On chronic exposure, individuals present with memory loss, neurological problems, insomnia, psychosis, tremors etc.

Patients with organophosphate poisoning usually present with  Salivation, Lacrimation, Urination, Defecation, Gastric cramps, Emesis (SLUDGE) symptoms.

44. A 38-year-old female suddenly developed acute inflammatory rash in form of roseolas, papules, vesicles that are scattered on the skin of trunk in irregular and predominantly focal manner. The rash appeared a few hours after visiting a restaurant. The patient complains of itching skin. What is the most likely diagnosis?

Explanation

Toxicodermatosis is a disease of the skin caused by a poison or by an allergen. The causative agents may be such chemical substances as antibiotics, sulfanilamides, and bromine, iodine, arsenic, and mercury compounds, or such food products as berries, cheese, or fish. As described in this scenario, the rash appeared after visiting a restaurant. Toxicoderma can also occur as a metabolic disorder or an occupational disease.

The causative agent enters the body through the respiratory organs or the digestive tract, during injections, or by direct contact with the skin. The primary manifestation of toxicoderma is a rash, which is generally symmetrical; it can take the form of inflamed or hemorrhagic maculae, papule, or vesicles (blisters). In severe cases, widespread areas of the skin are affected (erythroderma) and the patient manifests fever, cardiovascular disorders, or symptoms of anaphylactic shock.

With the removal of the allergen the symptoms of toxicoderma rapidly disappear, although relapses (usually severe) may occur with subsequent repeated exposure to the causative agent. Treatment consists in avoiding contact with the suspected allergen or toxic agent, as well as in the use of desensitizing agents and of expectant treatment.

Atopic dermatitis is a chronic condition that requires symptom management. It’s characterized by an itchy, red rash that usually appears at joints in your body, such as knees or elbows, and even around the neck. This condition occurs in flare-ups or bouts, meaning that it gets worse and improves in irregular cycles.

Many doctors use the terms “atopic dermatitis” and “eczema” interchangeably. They are both generic terms for “inflammation of the skin.” Eczema usually starts in early infancy, but also affects a substantial number of adults. It is commonly associated with elevated levels of immunoglobulin E (IgE) and a series of allergic diseases—including food allergy, asthma, and allergic rhinitis.

Contact Dermatitis occurs when your skin has a reaction to something it’s come in contact with. This can include bleach, soap, poison ivy, certain metals, or other irritants. The rash is typically red and may itch or burn.

 
45. Public nurseries are designed as a single or several one- or two-storey buildings linked by covered walkways. Planning the structure of preschool instiutions is based upon the following principle:

Explanation

46. A 50-year-old male in a grave condition has been admitted to the intensive care unit. It is known from life history that the patient works in agriculture, and 3 hours ago was engaged into insecticide treatment of crops for control of colorado potato beetle. Condition on admission: acrocyanosis, bronchorrhea, tachypnea, AP- 100/60 mm Hg, Ps- 44/min. What method of efferent therapy would be most appropriate at this stage?

Explanation

Hemosorbtion: Removal of toxins or metabolites from the circulation by the passing of blood, within a suitable extracorporeal circuit, over semipermeable microcapsules containing adsorbents (e.g., activated charcoal) or enzymes, or other adsorbents (e.g., various resins, albumin-conjugated agarose).

Hemodialysis: A medical procedure to remove fluid and waste products from the blood and to correct electrolyte imbalances. This is accomplished using a machine and a dialyzer, also referred to as an "artificial kidney."

Hemodialysis is used to treat both acute (temporary) and chronic (permanent) kidney failure.

Plasmapheresis (PP) is the removal of whole blood from the patient, its separation by machine into component parts, and then the return of certain of those components to the patient.  Indications include Gulliane-barre syndrome, good pasture syndrome, chronic demyelinating disorders , Thrombotic thrombocytopenic purpura etc.

 
47. Carpathian region is characterized by permanently high (over 80%) air humidity. In the cold season the population of this region feels very cold at moderately low temperatures. This is due to an increase in the heat transfer by:

Explanation

Convection refers to the transfers of heat via the movement of fluids and gases. Note that relative humidity also has a direct effect on convective heat transfer. Higher humidity will raise the amount for convective heat transfer coefficient and subsequently the rate of heat transfer increases.

Conduction involves the direct transfer of heat from one molecule to another. 

Radiation. Radiation involves the transfer of heat through the air or a vacuum. Heat from the sun is carried by radiation. The human body radiates heat in all directions. The ability to dissipate body heat by radiation depends on the temperature of the environment. Environmental temperature must be less than that of the body for heat loss to occur.

 
48. An employee has been diseased for 4 months, further treatment is necessary, the patient is incapacitated. Who is authorized to provide further disability examination of this patient?

Explanation

49. A factory’s sectorial doctor selects a group of persons who often fall ill for thorough monitoring. At the same time he takes into consideration the number of etiologically related cases with temporary disability in each of the employees over the last year. An employee falls into this group if the number of sickness cases is:

Explanation

50. A family lives in the town situated within the zone of radiation pollution. A 6-year-old child had been ill with ARVI for 19 days. The child was undergoing outpatient treatment and was nursed by his mother, a cafe worker. Specify the order of disability examination:

Explanation

Employees are entitled to emergency time off (sick leave) to care for a dependant.

A dependant includes:

  • spouse/partner,

  • child/grandchild,

  • parent/grandparent or

  • someone else who depends on you for their care.

 

Your employer may ask for evidence that you are entitled to parental leave. A birth certificate will usually do the trick and you’re entitled to return to the same job after taking parental leave.

51. It is planned to organize a rural outpati- ent clinic. The patients will be able to visit the doctors of the following specialities:

Explanation

52. A 12-year-old girls has minor functi- onal and morphological abnormalities: 1,0 D myopia, reduced body resistance. The patient has no history of chronic diseases. Over the last year, there were 4 cases of respiratory di- seases. The girl belongs to the following health group:

Explanation

53. A hospital nutrition unit received a batch of beef. Sanitation physician examined the meat and revealed the presence of 5 bladder worms per 40 cm2 of meat. Give the hygienic assessment of meat:

Explanation

 

Due to the presence of bladder worms (parasitic worms) in the meat, the meat should not be consumed. It should be technically (properly) disposed of.

54. An emergency physician arrived to provide medical care for a hangman taken out of the loop by his relatives. The doctor revealed no pulse in the carotid arteries, lack of consciousness, spontaneous breathing and corneal reflexes; cadaver spots on the back and posterior parts of extremities. A person can be declared dead if the following sign is present:

Explanation

Forensic pathologists are essentially required to estimate the time since death (TSD) to assist in death investigation. The early post-mortem phase is most frequently estimated using the classical triad of post-mortem changes – rigor mortis, algor mortis and livor mortis. 

Algor Mortis: Humans are warm-blooded organisms, which means that we maintain a constant internal temperature, regardless of the outside environment. The brain is our thermostat and the circulatory system is the main heat dissipator. However, within seconds of death, the brain cells begin to die and the heart stops pumping blood. Without the brain and and the blood distributing heat, the corpse eventually starts to match the outside temperature.

Rigor mortis is the post-mortem stiffening of muscles, caused by the depletion of adenosine triphosphate (ATP) from the muscles.

 

Livor Mortis: This is the final stage of death. When the heart stops beating, the blood is now at the mercy of gravity. It tends to collect at certain parts of the body. Depending on the position of the body, these parts would vary. For instance, if the person was flat on their back when they died, the blood would collect in the parts that are touching the ground (just as in this patient - cadaver spots were on the back and posterior parts of the extremities). If the person was hanging, it would collect in their fingertips, toes, and earlobes. Lividity starts with the skin where the blood has settled developing a bright red color. After a few hours, the color changes from red to bluish-purple. The bluish coloring of the skin is called livor mortis or cadaver spots.

55. A 69-year-old male patient has been hospitalized with hypothermia. Objectively: the patient is pale, has shallow breathing. AP- 100/60 mm Hg, Ps- 60/min. Palpation of the abdomen and chest reveals no pathological signs. The body temperature is of 34.8*C. The patient’s breath smells of alcohol. Give treatment recommendations:

Explanation

Hypothermia is a severe condition in which the body temperature drops to an abnormally low level. It occurs when the body is unable to produce enough heat to counter the heat that it is losing. Under healthy conditions, the body maintains a relatively stable temperature of around 98.6˚F or 37˚C. 

The following techniques can help treat hypothermia:

Passive external rewarming: This uses the individual’s heat-generating ability. It involves removing their cold, wet clothing, ideally replacing it with adequately insulated, dry clothing, and moving them to a warm environment. 

Active external rewarming: This involves applying warming devices, such as hot-water bottles or warmed forced air, externally to truncal areas of the body. For example, the individual could hold a hot-water bottle under each arm.

Active core rewarming: This uses warmed, intravenous fluids to irrigate body cavities, including the thorax, peritoneum, stomach, and bladder. Other options include getting the individual to inhale warm, humidified air, or applying extracorporeal rewarming by using a heart-lung machine.

Do not give a person alcohol if they have signs of hypothermia, and avoid giving any drinks to an unconscious person.

 

Considering the critical condition of this patient, active core warming by using warmed intravenous fluids which gets into the system immediately and will bring the most desired result at the shortest time possible is by far the best treatment recommendation.

56. Examination of an electric welder with 15 years of service record revealed dry rales in the lower lung fields. Radiograph shows diffuse nodules sized 3-4 mm in the middle and lower lung fields. What disease can be suspected?

Explanation

The term coniosis refers to a group of diseases ( conditions) caused by the inhalation of dust ( of different forms). For Example Anthracosis ( pneumoconiosis) also called coal miners lung occurs as a result of inhalation of coal or carbon. Silicosis ( grinder’s disease) -  inhalation of crystalline silica dust.  Siderosis- Iron inhalation. Berylliosis- Beryllium, Byssinosis - cotton etc. These mineral dusts when inhaled over time accumulate and deposit in the lung tissues forming infiltrates and fibrotic changes. 

 

This patient presents with a condition known as Pneumosiderosis ( welder’s lung) a chronic condition  caused by long term inhalation of iron dust; this condition is usually seen in welders.

57. Evaluation results of sanitary and hygiene conditions in a 4-bed ward were as follows: ward area - 30 m2, height - 3,2 m, temperature - 20o C , humidity - 55%, air velocity - 0,1 m/s, window-to-floor area ratio - 1:5, daylight ratio - 0,6%, concentration of carbon dioxide in the air - 0,1%. Which of the given indicators does not meet hygienic requirements?

Explanation

58. The outpatient department of a city hospital works also as a 60-bed day hospital for somatic patients. The department operates in a single-shift mode. What specialist selects patients for admission to the day hospital?

Explanation

59. Within the structure of the population the share of persons aged 0 to 14 years is 25%, the share of persons aged 50 years and older is 30%. What concept most accurately describes this demographic situation?

Explanation

 

A regressive type of population is one in  which the percentage of older individuals is higher than that of the early aged (younger). In the Progressive type, the younger ones are more in the population. Notice that in this analysis, the percentage of persons aged 50 and above is greater than the percentage of the individuals aged 0-14.

60. During examination of a patient, the doctor detected in him disorders of the eyes (hemeralopia, Bitot\'s spots), skin and skin appendages, mucosa, and gastrointestinal tract. He was provisionally diagnosed with Prasad\'s syndrome. What causes the development of this pathology?

Explanation

Prasad Syndrome is a condition characterised by geophagia ( the habit of eating soil, clay etc), hypogonadism, growth retardation and zinc deficiency. Bitot spots are keratin fragments found/deposited on the conjunctiva; they are usually found due to Vitamin A deficiency. Zinc deficiency may also be involved with the pathogenesis of secondary vitamin A deficiency. Inadequate zinc can depress the hepatic synthesis of retinol-binding protein (RBP), which is required for mobilization of retinol from the liver. In addition, zinc may play a role in the conversion of beta-carotene to retinol via the enzyme 15-15 dioxygenase. 

In iron deficiency, iron deficiency anemia will be observed (brittle nails, soreness in the tongue, weakness, pale skin etc). Copper deficiency is seen in Menkes disease.

 
61. A 37-year-old woman received an occupational trauma that resulted in a severe vision impairment. Now she needs to be trained for another occupation. What type of rehabilitation should the doctor choose for the patient in this case?

Explanation

Occupational/ professional Rehabilitation involves specialized healthcare dedicated to improving, maintaining and restoring the physical strength, cognition and mobility with maximum results. It helps an individual to recover from addictions, injuries and disorders.

Social rehabilitation — a set of activities aimed at rehabilitation and improving the functional capability of people and their inclusion in the society.

Psychological rehabilitation focuses on restoring one’s mental status.

 
62. A 28-year-old man, a teacher, after an emotional stress developed painful muscle spasms in his right hand that occur during writing; now he has to hold the pen between the second and third fingers. He has no problems with typing or writing on the blackboard; no other motor disturbances or neurological pathologies are detected. What is the most likely diagnosis?

Explanation

Writer’s cramp is a specific form of focal dystonia that affects the fingers, hand, or forearm. Focal dystonia of the hands is a neurological movement disorder.

Cortical agraphia is a neurological disorder in which an individual loses the ability to communicate either through writing or he/she forgets how to spell.

 
63. The director of a medical facility draws up a financial plan for the next year. To improve the economic well-being of his establishment, he decided to increase the amount of medical services provided. How will it change the fixed cost per unit of service?

Explanation

64. 40% of the workers, who polish the art glass, using an abrasive disk, and have a long record of employment, are diagnosed with ulnar neuritis, 21% - with vegetative polyneuritis, and 12% - with vegetomyofascitis of the upper limbs. These pathologies are associated with the following harmful factor:

Explanation

An abrasive disc is a heavy equipment used in cutting and drilling jobs; from anamnesis, these workers have been using these tools for a long period of time and currently present with issues related to the nerves and muscles. These conditions are mainly due to the effect of long term exposure to vibrations from this work tool and environment.
65. After a long drive with the window open a man developed a facial asymmetry; he cannot close his right eye, his right nasolabial fold is smoothed out, movements of expression are absent on the right, there is a disturbance of taste sensation in the tongue on the right. No other neurological abnormalities were detected. What disease can be provisionally diagnosed in this case?

Explanation

Facial (nerve) palsy is a neurological condition in which function of the facial nerve (cranial nerve VII) is partially or completely lost.  Key findings include; Sensory disturbances : Painful sensation around or behind the ear, Impairment of taste in the anterior tongue , Hyperacusis; dropping of the mouth, dry mouth, Bell's phenomenon: a physiologic, reflexive movement of the eye (upward and outward) that occurs when the eyelid is actively closed, Lagophthalmos: The patient cannot fully close the eyes (due to paralysis of the orbicularis oculi muscle). Decreased lacrimation. 

In neuropathy of the trigeminal nerve ( trigeminal neuralgia), the individual will feel excruciating pain at the slightest pressure applied on the face e.g. while brushing the teeth, yawning etc.

In neuropathy of the oculomotor nerve, the individual will experience diplopia and strabismus. 

 
66. A 30-year-old woman made an appointment with the family doctor for scheduled vaccination of her 2-year-old child. What type of healthcare provides such medical services?

Explanation

According to levels of specialization, types of medical care include: Emergency, Primary, Secondary, Tertiary, palliative, medical rehabilitation and dentistry.

Primary medical care: involves care given by nurses, midwives, general physicians, family doctors, in polyclinics etc. One major aspect of primary health care is preventive medicine - these include vaccination, teaching on methods for good living etc.

Secondary medical care: When your primary care provider refers you to a specialist, you are then in secondary care. Secondary care simply means you will be taken care of by someone who has more specific expertise eg cardiologist, endocrinologist etc

Tertiary Medical Care: Once a patient is hospitalized and needs a higher level of specialty care within the hospital, he may be referred to tertiary care. Tertiary care requires highly specialized equipment and expertise. 

 
67. A patient is 45 years old. He was referred for a consultation with a psychiatrist due to complaints of abdominal pain and discomfort that occur in emotionally straining situations. Objectively no changes of the gastrointestinal tract were detected. The complaints emerged over 10 years ago against the background of severe alcohol poisoning. The patient has been repeatedly visiting gastroenterologists, who were unable to find any significant changes in the patient. The prescribed therapy was ineffective. What is the likely conclusion?

Explanation

In Somatoform Autonomic dysfunction, the symptoms are presented by the patient as if they were due to a physical disorder of a system or organ that is largely or completely under autonomic innervation and control, i.e. the cardiovascular, gastrointestinal, respiratory, and  urogenital systems. Clinical and instrumental examination revealed no organic alterations in any system therefore indicating a somatoform autonomic dysfunction. 

Organic brain syndrome is defined as a state of diffuse cerebral dysfunction associated with a disturbance in consciousness, cognition, mood, affect, and behavior in the absence of drugs, infection, or a metabolic cause

 
68. Forensic autopsy of the body of a 59-year-old man, who died suddenly at home without signs of violent death, shows pink skin and mucosa, liquid bright-red blood, and bright-red plethoric internal organs. Forensic toxicology testing detected 1.44°/00 of ethanol in the blood and carboxyhemoglobin levels of 55%. What is the cause of death?

Explanation

 

Notice that the carboxyhemoglobin level is 55% (very high) - this indicates poisoning by carbon monoxide; the level of ethanol found in his blood is too little to cause poisoning or be the major reason behind his death.

69. For 20 years the role of excessive weight in ischemic heart disease development among the working age male population over 40 was studied. It was determined that overweight men developed ischemic heart disease more often. What type of epidemiological study is it?

Explanation

 

Cohort study refers to a cross section of interval through time; Whereas the cohort study is concerned with frequency of disease in exposed and non-exposed individuals, the case-control study is concerned with the frequency and amount of exposure in subjects with a specific disease (cases) and people without the disease (controls).

Simply put:

In cohort study - both groups will have Ischemic Heart Disease, then you consider a risk factor that might have led to the Ischemic Heart Disease. In this question, you are studying overweight and normal weight in those with Ischemic Heart Disease and the study shows that overweight men developed ischemic heart disease more often.

In case control - you will compare those with Ischemic heart disease (Cases) and those without Ischemic Heart Disease (Control).

70. In a pediatric clinic, located in a rural area, there are 9 children, who simultaneously fell ill. The following signs were detected: low physical activity, acrocyanosis of the nasolabial triangle and fingertips, mucosal cyanosis, tachycardia, dyspnea. It was determined that all the sick children were fed with a formula that was dissolved in the water taken from a dug well. Laboratory analysis revealed high levels of methemoglobin in the blood of the children. These signs can be caused by increased content of a certain element in the water. Name this element:

Explanation

Methemoglobinemia is an unusual and potentially fatal condition in which hemoglobin is oxidized to methemoglobin and loses its ability to bind and transport oxygen. The most common cause of methemoglobinemia is the ingestion or inhalation of oxidizing agents such as nitrates or nitrites. Signs include dizziness, headaches, cyanosis, dyspnea , tachycardia etc. Treatment of this condition involves the use of methylene blue and vitamin C; other substances that can cause this state (methemoglobinemia) include dapsone and benzocaine. 

Lead poisoning is characterised by lead lines on the gingiva (burton lines), encephalopathy, erythrocyte basophilic stippling, abdominal colic, sideroblastic anemia and wrist or foot drop. Dimercaprol ( chelating agent) is used in treatment.

 
71. When planning treatment of a patient, it was decided to use a medicine with evidence level A. What trials produce the evidence that allows to classify the medicine as level A?

Explanation

72. A 57-year-old man, a miner, complains of a pain in his chest, dyspnea on physical exertion, excessive sweating, constant subfebrile tempeature, and cough that produces blood-streaked sputum. He has been smoking for approximately 40 years (2 packs a day) and frequently has ’’pneumonias” Survey chest X-ray shows a triangular shadow in the middle lobe of the right lung. One of the apices of the shadow points to the lung root. Cardiac and mediastinal shadows are displaced toward the affected area. Make the provisional diagnosis:

Explanation

73. A district doctor has diagnosed one of his patients with dysentery What accounting document reflects this type of morbidity?

Explanation

74. What modern organzational method can provide the patients in the remote settlements with timely access to quality medical aid and such medical services as consulting, diagnostics, and treatment, especially in the situations when time and distance are crucial?

Explanation

 

Telemedicine is the exchange of medical information from one location to another using electronic communication, which improves patient health status. It involves remote diagnosis and treatment of patients by means of telecommunications technology.

75. A man works in casting of nonferrous metals and alloys for 12 years. In the air of working area there was registered high content of heavy metals, carbon monoxide, and nitrogen. During periodic health examination the patient presents with asthenovegetative syndrome, sharp abdominal pains, constipations, pain in the hepatic area. In urine: aminolevulinic acid and coproporphyrin are detected. In blood: reticulocytosis, low hemoglobin level. Such intoxication is caused by:

Explanation

Aminolevulinic acid and coproporphyrin are amino acids necessary for the synthesis of Heme. The excretion of these constituents in urine indicates either an inherited or secondary Porphyria. One of the most frequent causes of the secondary form of this disease is Lead poisoning. An increased degradation of porphyrin will lead to a decrease in Hemoglobin synthesis. Also note that the patient experiences Asthenovegetative syndrome ( physical and nervous fatigue), pain in the hepatic area and stomach; these are classical signs of Lead poisoning.

Carbon monoxide poisoning will be characterised by headache, lethargy, dizziness, loss of consciousness etc.

 
76. On laboratory investigation of a pork sample there is 1 dead trichinella detected in 24 sections. This meat should be:

Explanation

 

This risk of getting Trichinellosis is very high if this meat is consumed; this piece should be technically disposed.

77. Employees work in conditions of high dust concentration. Certain chemical (silicon dioxide content) and physical properties of dust aerosols contribute to the development of occupational dust-induced diseases. What is the main physical property of dust aerosols?

Explanation

Dispersion is the separation or distribution of a given substance over a particular area. Dust is easily carried by and spread around by air.

Solubility refers to the ability of a given substance (the solute) to dissolve in another substance (the solvent).

Ionization refers to any process by which electrically neutral atoms or molecules are converted to electrically charged atoms or molecules (ions).

 
78. A 39-year-old man, a battery attendant, suddenly developed weakness, loss of appetite, non-localized colicky abdominal pains, and nausea. Objectively his skin is gray; there is a pink-gray stripe on his gums; the abdomen is soft and sharply painful. Blood test detected erythrocytes with basophilic stippling and anemia. The patient has a history of peptic ulcer disease of the stomach. Constipations occur each 3-4 days. What is the most likely provisional diagnosis?

Explanation

 

Lead inhibits ferrochelatase and ALA dehydratase which leads to a decrease in Heme synthesis and increased RBC protoporphyrin. Lead also inhibits rRNA degradation which results in RBC’s retaining aggregates of rRNA (seen as basophilic stippling). Symptoms of lead poisoning include; Lead lines on gingiva (Burton lines) and on metaphysis of bones ( on x-ray), Encephalopathy and erythrocyte basophilic stippling, Abdominal colic and sideroblastic anemia, wrist and foot Drop. A chelating agent such as Dimercaprol is used as a first line of treatment. 

79. During regular medical examination a lyceum student presents with signs of cheilitis that manifests as epithelial maceration in the area of lip seal. The lips are bright-red, with single vertical cracks covered with brown-red scabs. These clinical signs are most likely caused by insufficient content of the following in the diet:

Explanation

The signs and symptoms of riboflavin (vitamin B2) deficiency typically include sore throat with redness and swelling of the mouth and throat mucosa, cheilosis and angular stomatitis (cracking of the lips and corners of the mouth), glossitis, seborrheic dermatitis or pseudo-syphilis, and a decreased red blood cell count with normal cell size and hemoglobin content (normochromic normocytic anemia).

Thiamine (Vit B1) deficiency is known as Beri-beri, deficiency of ascorbic acid will lead to scurvy, easy bruising and bleeding are seen in such patients.

Calciferol (Vit D): Rickets in Children; Osteomalacia in Adults

Ascorbic Acid (Vit C): Scurvy - bleeding gums, easy bruising

Retinol (Vit A): Night blindness

 
80. Children from a certain township present with brittle teeth, malocclusion, dental enamel erosions, and dental pigmentation that looks like yellow-brown spots. What is the likely cause of this presentation?

Explanation

 

Presence of dark-yellow pigmentation of the tooth enamel, diffuse osteoporosis of bone apparatus, ossification of ligaments and joints are specific symptoms that indicate Fluorosis. This is a condition that occurs as a result of excessive intake of fluorine.

81. An unconscious patient was delivered to a hospital by an ambulance. Objectively his body temperature is 39°C, he presents with convulsions and red dry skin. It is known that the patient works as a stoker in the boiler room. What is the likely diagnosis?

Explanation

From anamnesis, we observe that this patient works in a very hot/steaming environment; the signs shown by this patient i.e. a red dry skin and a temperature of 39℃ indicates the effect of heat on the patient.

Heat stroke is a form of hyperthermia in which the body temperature is elevated dramatically. The cause of heat stroke is an elevation in body temperature, often accompanied by dehydration.

The most common symptoms of CO (carbon monoxide) poisoning are headache, dizziness, weakness, upset stomach, vomiting, chest pain, and confusion on the background of inhalation of fumes from cars, generators etc.

Acute food poisoning will often present with vomiting, nausea and diarrhea.

 
82. A 56-year-old woman has been working as a disinfector for 19 years. She complains of general weakness, nausea, bitter taste in her mouth, heavy sensation in her right subcostal area, and rapid fatigability Objectively her body temperature is 37°C, the sclerae are icteric, and the liver is enlarged, total bilirubin is 40 mcmol/L. What is the likely diagnosis?

Explanation

83. A middle school teacher with 4-year-long record of work was issued a medical certificate for pregnancy and childbirth leave. What amount of pay will she receive for the duration of her leave in this case?

Explanation

84. In the inpatient gynecological unit within a year 6500 women underwent treatment. They spent there a total of 102000 bed-days. What indicator of the gynecological unit work can be calculated based on these data?

Explanation

85. At the railroad crossing a passenger train collided with a bus. In this collision 26 bus passengers died, another 18 passengers received mechanical injuries of varying severity. Where will be professional medical aid provided for the victims of this accident? Who will provide this aid?

Explanation

86. A 42-year-old man, a dispatcher, suffers from peptic ulcer disease of the duodenum. The disease is of moderate severity. He wants to be assigned a disability group. Make the conclusion regarding his working ability:

Explanation

87. In the air of the feed kitchen at the poultry factory, at the area where formula feed is being mixed, the dust concentration reaches 200 mg/m3. Air microflora is represented predominantly by Asperqillus and Mucor fungi. What effect determines pathogenic properties of the dust?

Explanation

88. A child is 1 year old. After solid food was introduced into the diet, within the last several months the child developed loss of appetite, diarrhea with large amount of feces, and occasional vomiting. Body temperature remains normal. Body weight is 7 kg. The child is very pale, has leg edemas and extremely distended abdomen. Feces analysis detects high levels of fatty acids and soaps. Diagnosis of celiac disease was made and gluten-free diet was prescribed. What should be excluded from the diet in this case?

Explanation

Celiac disease is an immune reaction to eating gluten, a protein found in wheat, barley and rye. Sometimes called celiac sprue or gluten-sensitive enteropathy. When people with celiac disease eat gluten (a protein found in wheat, rye and barley), their body mounts an immune response that attacks the small intestine. These attacks lead to damage on the villi, small fingerlike projections that line the small intestine, that promote nutrient absorption. When the villi get damaged, nutrients cannot be absorbed properly into the body.

Celiac disease is hereditary, meaning that it runs in families. People with a first-degree relative with celiac disease (parent, child, sibling) have a 1 in 10 risk of developing celiac disease.

Celiac disease can develop at any age after people start eating foods or medicines that contain gluten.

 

Oats contain avenin, which is a protein similar to gluten. Research has shown that most people with coeliac disease can tolerate gluten free oats with no problems. The issue is that sometimes oats are produced in the same place as wheat, barley and rye, and then become contaminated with these other grains.

89. A regional cardiologist is given a task to develop a plan for preventive measures aimed at decreasing cardiovascular mortality rates. What measures should be planned for secondary prevention?

Explanation

Primary prevention of a disease is aimed at preventing the onset (start of a disease) i.e., the disease is yet to occur so we aim at preventing it from happening via optimization of lifestyle and living conditions etc.

Secondary prevention is aimed at preventing the progress of a disease (early diagnosis and treatment) i.e., the patient already has the disease, so we are more concerned on stopping it from progressing and preventing its complications.

 
90. A 10-year-old boy, who was outdoors in windy and cold weather, developed moderate pain and tingling in his fingers and toes. When he had returned home, his parents noticed that the tips of his fingers and toes were white and their sensitivity was lost. The affected areas are warming up, the fingers are tingling and in pain. Skin pallor changed into redness, tingling stopped, slight itching and swelling of the fingers appeared. Determine the frostbite degree in this child:

Explanation

Frostbite is an injury of the skin and underlying tissues that occur due to exposure to cold ( low temperature). There are 4 degrees of frostbite:

I Degree: lasts for about 5-7 days; after warming, paleness changes to hyperemia (redness). Edema of tissues progresses for about 2 days and then it decreases to 6-7 days when shelling (peeling) of epidermis appears. Tactile and pain sensitiveness (sensation) are preserved but sometimes with disorders.  Pain in injured areas could be severe, itching also could be.

II Degree: characterised by the spreading of edema, appearance of bullaes. The bottom of the opened bullaes are covered with fibrin. Cyanotic skin and difficulty of movement of phalanges. Necrosis of keratic and granular layers. N/B growth layer of the skin is not lost and regeneration appears after 2 weeks. Scars are not formed.

III Degree: Necrosis of all skin layers or even fatty tissue appears. Inflammation develops: firstly aseptic and then on 5th- 7th day purulent . Bullas contain blood . Decrease of tactile and temperature sensation. Edema of tissues spreads on the proximal areas. Firstly skin has cyanotic color then dark brown and black crusts are formed.

IV Degree: Necrosis of all skin, fat tissue and even bones and joints. Results in amputation of the affected area.

 
91. Caries morbidity rate is 89% among residents of a community. It is determined that fluorine content in water is 0.1 mg/L. What preventive measures should be taken?

Explanation

 

Dental caries is a bacterial disease that begins with demineralization of the outermost dental enamel and progresses, if not halted, can lead to loss of tooth substance and infection of the dental pulp. Fluoride acts in several ways to prevent caries. The principal action is thought to be that fluoride in dental plaque inhibits the initial demineralization of enamel, and then promotes remineralization of early lesions. Fluorine is gotten from drinking water and food materials. Adequate amount of fluorine in water should be from 0.7-1.5

92. A 39-year-old man, a battery attendant, suddenly developed weakness, loss of appetite, nonlocalized colicky abdominal pains, and nausea. Objectively his skin is gray; there is pink-gray stripe on his gums; the stomach is soft and sharply painful. Blood test detected erythrocytes with basophilic stippling and anemia. The patient has a history of peptic ulcer disease of the stomach. There is tendency to constipation. What is the most likely provisional diagnosis?

Explanation

Lead inhibits ferrochelatase and ALA dehydratase which leads to a decrease in Heme synthesis and increased RBC protoporphyrin. Lead also inhibits rRNA degradation which results in RBC’s retaining aggregates of rRNA (seen as basophilic stippling). Symptoms of lead poisoning include; Lead lines on gingiva (Burton lines) and on metaphysis of bones ( on x-ray), Encephalopathy and erythrocyte basophilic stippling, Abdominal colic and sideroblastic anemia, wrist and foot Drop. A chelating agent such as Dimercaprol is used as a first line of treatment.
93. The inpatient surgery unit has introduced the method of laparoscopic cholecystectomy into its practice. As the result the average duration of postoperative care provided to the patients could be reduced to 3.4±0.8 days compared to 7.3±1.1 days that were required after non-laparoscopic cholecystectomy. What method of medical statistics can confirm the statistical significance of the difference between these two estimates?

Explanation

94. A 37-year-old worker during a fire ended up in the area of high CO concentration. He was delivered to a hospital in unconscious state. Objectively: the skin of his face and hands is crimson. Respiration rate is 20/min. ECG: alterations specific for hypoxic myocardium. Hourly diuresis is 40 ml. Blood test: erythrocytes -4.5·1012/L, Нb- 136 g/L, color index - 0.9, ESR-3 mm/hour, carboxyhemoglobin - 5%. What criterion allows determining the severity of the patient’s condition?

Explanation

 

The patient who is currently expressing signs of  excess carbon monoxide exposure (unconscious state, discoloration of skin of the hands and face, ECG changes indicating high levels of hypoxia) has normal hemoglobin concentration and color index but a high amount of carboxyhemoglobin ( 2-3% norm).  Recall that hemoglobin binds to oxygen forming oxyhemoglobin; in this form, oxygen is transported to various parts of the body. In the presence of Carbon (CO₂ or CO), oxygen is preferentially discharged since carbon has a higher affinity for hemoglobin; carbon then binds to hemoglobin (in this case carbon monoxide forming carboxyhemoglobin) thereby reducing the oxygen carrying capacity of hemoglobin. Hypoxia (reduced oxygen supply) to the brain will lead to a hypoxic coma.

95. In April during the medical examination of various population groups, 27% of individuals presented with low working ability and rapid fatigability. The following symptoms were observed in the affected individuals: swollen friable gingiva that bleeds when pressed, hyperkeratosis follicularis not accompanied by skin dryness. These symptoms most likely result from the following pathology:

Explanation

Vitamin C is needed for the regulation of collagen synthesis (it stabilizes collagen mRNA, thereby increasing collagen synthesis). In its absence, collagen synthesis is unstable leading to reduced wound healing and strength of tissues; symptoms include frequent gingival and petechial hemorrhages as seen in the patient. Vitamin C deficiency is known as Scurvy.

In Vit A deficiency, the patient will experience impaired vision especially at night; this is because Vit A is a precursor of rhodopsin- the photo pigment found in rods (responsible for black and white vision).

Vit B1 deficiency leads to Beri Beri

 
96. A man works in casting of nonferrous metals and alloys for 12 years. In the air of working area there was registered high content of heavy metals, carbon monoxide, and nitrogen. During periodic health examination the patient presents with asthenovegetative syndrome, sharp pains in the stomach, constipations, pain in the hepatic area. In urine: aminolevulinic acid and coproporphyrin are detected. In blood: reticulocytosis, low hemoglobin level. Such intoxication is caused by:

Explanation

Aminolevulinic acid and coproporphyrin are amino acids necessary for the synthesis of Heme. The excretion of these constituents in urine indicates either an inherited or secondary Porphyria. One of the most frequent causes of the secondary form of this disease is Lead poisoning. An increased degradation of porphyrin will lead to a decrease in Hemoglobin synthesis. Also note that the patient experiences Asthenovegetative syndrome (physical and nervous fatigue), pain in the hepatic area and stomach; these are classical signs of Lead poisoning.

Carbon monoxide poisoning will be characterised by headache, lethargy, dizziness, loss of consciousness etc.

 
97. Due to introduction of a new treatment method, average duration of therapy in the experimental group was 12.3±0.2 days compared to 15.4±0.4 days in the control group that was treated by the old method. What calculations should be made to estimate the statistical significance of the difference in the results?

Explanation

The T-test is a statistical, hypothetical test used to compare the mean (average) of two related variables. For example comparing the mean of the experimental group to that of the controlled group. It lets you know how significant the differences are.

The Z-test also compares the means of two populations although it relies on the Variance ( the square root of the mean deviation).

 
98. Establishments participating in medical examinations include: medical and preventive treatment facilities, hygiene and preventive treatment facilities, sociomedical expert committees, Ministry of Defence medical committees, Ministry of Domestic Affairs medical committees, forensic medicine agency, etc. Specify what service deals with sociomedical assessment of temporary disability:

Explanation

 

Temporary disability refers to a disability that occurs for a short time period. The medical and preventive treatment facilities are responsible for the assessment and care of such individuals.

99. In the process of hiring, a prospective employee has undergone preventive medical examination and was declared fit to work in this manufacturing environment. What type of preventive medical examination was it?

Explanation

100. An employee has been sick for 4 months, further treatment is necessary, the patient is unable to work. Who is authorized to provide further disability examination of this patient?

Explanation

The right of prolongation a medical certificate of invalidity alongside with the medical treatment by the doctor is granted by:

• Chief medical officer (assistant of the chief medical officer on expertise of ability for work);

• Manager of department;

• Medical-consulting commission also known as medical expertise committee

 

The medical-consulting commission (MCC) is formed in case the doctors staff is above 15 doctors.

101. A population of a small town often presents with registered cases of juvenile cardiomyopathy, atherosclerosis, hypertension, endocrinopathy, chronic dermatitis, and arthralgia - signs of Keshan disease. What is the most likely cause of this pathology?

Explanation

 

Keshan’s disease is an endemic cardiomyopathy caused by a dietary deficiency of selenium and can be triggered by a mutated strain of the Coxsackie virus; the disease is characterised by heart failure and pulmonary edema and increases the chances of having cancer, hypertension and cardiovascular related issues. It is corrected by administering  Selenium supplements.

102. To assess the effectiveness of medical technologies and determine the power and direction of their effect on the public health indicators, the research was conducted to study the immunization rate of children and measles incidence rate by district. What method of statistical analysis should be applied in this case?

Explanation

103. The objective of a statistical research was to find out to what extent the population peruses the available medical services. For this purpose 300 residents of the area were interviewed. Information was collected by means of a special questionnaire. What method of collecting information was used by the researchers?

Explanation

 

The questionnaire passed across is a way of getting vital information. Anamnesis refers to the collection of previous/ existing information.

104. Estimation of community health level involved analysis of a report on diseases registered among the population of district under charge (reporting form 12). What index is calculated based on this report?

Explanation

105. The body of a 24-year-old woman with probable signs of poisoning has been found on the street. Forensic medical examination was requested by an investigator during examination of the site and the body. According to the criminal Procedure Code currently in force in Ukraine, forensic medical examination is required when it is necessary to determine the:

Explanation

According to the Criminal Procedure Code currently in force in Ukraine, forensic medical examination is required to determine the cause of death.
106. An excavator operator with 20 years of work experience at the opencast ore mine undergoes regular medical examination. He presents with signs of pneumoconiosis. What type of pneumoconiosis is the most likely in this case?

Explanation

 

Pneumoconiosis is a restrictive lung disease  caused by the inhalation of dust leading to fibrosis. Depending on the type of dust inhaled, different types exist. They include; Siderosis (iron ore), byssinosis (cotton), anthracosis (coal), Asbestosis (asbestos), silicosis (silica dust) etc.

107. A 52 year old man came to see his family physician complaining of pain in the chest. After taking history and performing physical exam the doctor decided to direct the patient to cardiologist for a consultation. What level of medical care is being proposed to the patient?

Explanation

According to levels of specialization, types of medical care include: Emergency, Primary, Secondary, Tertiary, palliative, medical rehabilitation and dentistry.

Primary medical care: involves care given by nurses, midwives, general physicians, family doctors, in polyclinics etc. Primary healthcare is the first contact a person has with the health system when they have a health problem. 

Secondary medical care: When your primary care provider refers you to a specialist, you are then in secondary care. Secondary care simply means you will be taken care of by someone who has more specific expertise eg cardiologist, endocrinologist etc

Tertiary Medical Care: Once a patient is hospitalized and needs a higher level of specialty care within the hospital, he may be referred to tertiary care. Tertiary care requires highly specialized equipment and expertise.

At this level, you will find procedures such as coronary artery bypass surgery, renal or hemodialysis, and some plastic surgeries or neurosurgeries.

 
108. A 30-year-old woman made an appointment with the family doctor for scheduled vaccination of her 2-year-old child. What type of healthcare provides such medical services?

Explanation

According to levels of specialization, types of medical care include: Emergency, Primary, Secondary, Tertiary, palliative, medical rehabilitation and dentistry.

Primary medical care: involves care given by nurses, midwives, general physicians, family doctors, in polyclinics etc. Primary healthcare is the first contact a person has with the health system when they have a health problem. For example, Immunizations/Vaccinations, health education on benefits of exercise, healthy diet etc., are some of the responsibilities of a primary health care system.

Secondary medical care: When your primary care provider refers you to a specialist, you are then in secondary care. Secondary care simply means you will be taken care of by someone who has more specific expertise eg cardiologist, endocrinologist etc

Tertiary Medical Care: Once a patient is hospitalized and needs a higher level of specialty care within the hospital, he may be referred to tertiary care. Tertiary care requires highly specialized equipment and expertise.

 

109. A 38-year-old woman has been working as a milker for 15 years. She made an appointment with the doctor due to development of red rashes on her hands, predominantly in the interdigital space. The rashes are weeping, itching, and expanding on her skin. Examination of her hands shows her nail plates to be yellow and brittle. These presentations aggravate during work. Make the provisional diagnosis:

Explanation

Eczema (dermatitis ) is a chronic inflammatory disease of the skin characterized by the presence of red, itchy, dry scaly rashes. Occupational eczema (dermatitis) is gotten from contact with certain chemicals in the course of working. The patient in question is a milker ( a person who extracts milk from cows) and most likely gets dermatitis as a result of contact with these animals or certain chemicals.

N/B Scabies is caused by a mite Sarcoptes scabiei; pemphigus is an autoimmune disease (antibodies against desmoglein), sores and blisters are seen in the mouth and genitals.

 
110. 40-50 minutes after the completion of repair works conducted in a closed garage, with car engine running, the repair workers developed severe headache in the temporal area, nausea, tinnitus, vertigo, etc. These symptoms are characteristic of acute poisoning with:

Explanation

 

Note that the man works in a closed garage and carbon monoxide is the type of gas released from the engines of these vehicles. Accumulation of these gases is poisonous for health and leads to the expressed symptoms.

111. A 37-year-old man working as a typesetter in a print shop complains of rapid fatigability, paroxysmal attacks of stomachache, weak drooping hands. Examination of neurological status revealed hypotrophy of the forearm muscles. Carporadial reflexes are sharply weakened. Sensitivity is not disturbed. Gums present with dark blue border. What neurological pathology is it?

Explanation

The patient presents with weak dropping hands and a weakened carporadial reflex; this sign is often seen in individuals poisoned by lead (wrist and foot drop). Other major signs are ’ Lead lines’ on gingiva/gums (Burton's line), abdominal colic, encephalopathy and sideroblastic anemia.

Guillain-Barre syndrome is an autoimmune condition that destroys schwann cells via inflammation and demyelination of nerve fibres. It occurs as a result of molecular mimicry and triggered by infections, stress etc. major signs include Bilateral facial paralysis and respiratory failure.

Shingles is gotten from varicella zoster virus infection.

 
112. A 32-year-old pregnant woman at the term of 5-6 weeks was vaccinated against influenza along with her whole family. At that time she was not aware of her pregnancy. The pregnancy is wanted. The woman needs an advice from the family doctor regarding the maintenance of her pregnancy, namely whether there is a risk of fetal malformations because of received vaccination. What advice should the doctor give in this case?

Explanation

 

An inactivated influenza vaccine is reliably used as a form of prophylaxis from the influenza infection; optimal terms for vaccination are October-November. Influenza vaccination is indicated in children with: chronic bronchopulmonary diseases, cardiovascular diseases, hemolytic anemias, diabetes mellitus, chronic kidney and liver diseases, HIV infection etc. Note that the flu vaccine is safe during pregnancy. The inactivated Influenza vaccine can be given to pregnant women at any trimester.

113. A healthy child 1 year and 5 months of age is being vaccinated against hepatitis B. The child did not receive the first dose of the vaccine previously, while in the maternity hospital. The doctor makes an individual vaccination schedule for this child and plans the administration of the next dose of the vaccine. What is the minimum interval between doses of vaccine in this case?

Explanation

 

A minimum interval is the shortest time between two doses in a vaccination series. The Hepatitis B vaccination is given in a three dose series; First dose at birth, second Between 1-2 months and the third dose is given between the 6th-15th month. Examples of the hepatitis B vaccines are Engerix-B and Recombivax HB; the minimum interval between the first and second doses is 4 weeks.

An adult taking the Hepatitis B vaccine will take 3 doses one month apart. For example, as an adult, if you take the first dose 1st March, next dose will be 1st April and the last dose will be 1st May.

114. A 45-year-old woman has been suffering from rheumatoid arthritis for 10 years and takes methotrexate twice a week. What statement regarding vaccination against pneumococci (23-valent vaccine) would conform to the recommendations for the management of rheumatoid arthritis issued by the European league Against Rheumatism in 2010?

Explanation

115. A 65-year-old woman was diagnosed with the following: chronic rheumatic heart disease, l degree of rheumatic activity; combined mitral heart disease with prevalence of III degree stenosis; heart failure IIA with retained left ventricular ejection fraction, functional class lII (NYHA). What tactics of vaccination against respiratory infections should be chosen to provide secondary prevention of exacerbations and to avoid heart failure decompensation in this patient?

Explanation

Vaccine-preventable diseases can increase the risk of cardiovascular complications. These infections are especially important in patients with heart disease: influenza; pneumococcal; tetanus, diphtheria, and acellular pertussis (Tdap); and zoster. Patients with heart diseases are advised to take a flu shot annually. It is also recommended that patients 65 and above with CVD should receive 1 dose of the 13-valent pneumococcal conjugate vaccine (PCV13) if they did not previously receive it. Another dose of PPSV23 should be administered at least 1 year after PCV13 and at least 5 years after the first dose of PPSV23.
116. A 60-year-old man presents with subcompensated viral liver cirrhosis (HCV), Child-Pugh class B. What tactics should be chosen regarding the vaccination against influenza in this case?

Explanation

Child Pugh classification is used to estimate the severity of liver diseases  according to the degree of ascites, the serum concentrations of bilirubin and albumin, the prothrombin time, and the degree of encephalopathy. A total Child-Turcotte-Pugh score of 5 to 6 is considered Child-Pugh class A (well-compensated disease); 7 to 9 is class B (significant functional compromise); and 10 to 15 is class C (decompensated disease). Two types of influenza vaccine are widely available: inactivated influenza vaccines (IIV) and live attenuated influenza vaccines (LAIV). IIV is approved for use in persons 6 months and older, including pregnant women and persons with chronic medical conditions. A flu vaccine is needed every season for two reasons. First, a person's immune protection from vaccination declines over time, so an annual vaccine is needed for optimal protection.
117. A 20-year-old student was brought to the first-aid center. He has a closed fracture of the left forearm and a contused lacerated wound on his left shin. After the patient received initial wound management, he presented the documents confirming that he has received all the necessary preventive vaccination as scheduled. What should the doctor do to prevent tetanus in this patient?

Explanation

 

Note that the patient presented the document that confirms he received all the necessary vaccination at the right time (which definitely includes a tetanus vaccine). In this case, the patient should be carefully monitored for the following few days in order to observe if any signs or changes that indicate a tetanus infection  occur.

118. A 40-year-old man developed fever up to 37.5*C and macular rash 10 days after the first dose of MMR (Measles-Mumps-Rubella) vaccine was administered. The vaccination was considered necessary as there was a measles outbreak in the city and the patient had not received MMR vaccination in his childhood. Is revaccination with MMR vaccine possible?

Explanation

Side effects from MMR vaccines that can occur 7 to 10 days after vaccination include:

  • fever (can be more than 39.4 °C), lasting two to three days

  • faint red rash (not infectious)

  • head cold, runny nose, cough or puffy eyes

  • drowsiness or tiredness

  • swelling of salivary glands

  • localised pain, redness and swelling at the injection site.

2 doses of the MMR vaccine provide the best protection against measles, mumps and rubella. The above listed side effects are expected in a small percentage of the population. Once the first dose is given, if any of the above listed side effects should occur, it is not a contraindication for the second dose as these side effects are expected. 

So, it is possible, as the patient in question, only had low grade fever and a macular rash.

However, it is important to note that uncommon and rare side effects like serious allergic reaction (anaphylaxis) might require a course of glucocorticoids treatment or given together with antihistamines. Thrombocytopenia, which is bleeding caused by insufficient blood platelets might be a contraindication for a revaccination. But that is not the case in this question.
119. A 26-year-old man is undergoing a regular check-up. One year ago he had a case of tonsillar diphtheria complicated with myocarditis. Presently his condition is satisfactory, no signs of cardiovascular failure; ECG shows first-degree atrioventricular block. What vaccine was administered to this man according to his age?

Explanation

This patient had Diphtheria a year ago but currently, his condition is satisfactory. The question wants to know which vaccine was given to him a year ago that helped in managing the disease.

D- Diphtheria; T- Tetanus; P- Pertussis (Whooping cough); aP- acellular pertussis vaccine.

 

It is recommended that diphtheria, tetanus, and acellular pertussis vaccination be administered across the lifespan. Children younger than 7 years of age receive DTaP or DT, while older children and adults receive Td. Therefore, in this patient who is older than 7 years (26 year old man), he was given a diphtheria and Tetanus Vaccine.

 
  • Give infants and children 5 doses of DTaP. Give one dose at each of these ages: 2 months, 4 months, 6 months, 15 through 18 months, and 4 through 6 years. 

  • Give adolescents a single dose of Tdap, preferably at 11 to 12 years of age.

  • Give pregnant women a single dose of Tdap during every pregnancy, preferably during the early part of gestational weeks 27 through 36. 

  • Give adults who have never received Tdap a single dose of Tdap. This can be given at any time, regardless of when they last got Td. This should be followed by either a Td booster every 10 years.

120. A patient received flame burns of both hands. On the dorsal and palmar surface of the hands there are blisters filled with serous fluid. The wrist joint region is hyperemic. The forearms were not injured. What is the provisional diagnosis?

Explanation

Rule of “nines” – area of different areas of the body is proportional: anterior surface of the trunk – 18 %, posterior – 18 %, lower limb – 18 %, external genitals – 1 %);

Rule of  “palm”. It is used if burns are limited and located on different areas of the body.

According to the rule of palm takes 1 % of the skin surface.

Division of the burns on superficial (I, II, IIIA st.) and deep (IIIB-IV st.)

I stage – hyperemia of the skin

II stage – separation of epidermis with formation of bullas

III A stage – necrosis of superficial layers of the skin with saving of bulbs hair, sweat glands and sebaceous glands.

IIIB stage – necrosis of all the derma

IV stage – necrosis of the skin and underlying tissues.  

Palmar surface - 1%; Dorsal surface - 1%, that's 2% for one hand and that will be 4% for both hands.

Blisters (bullae) - Stage II

121. During the periodic medical examination an assembly fitter (works on soldering details) didn’t report any health problems. Closer examination revealed signs of asthenic vegetative syndrome. Blood included red blood cells with basophilic aggregations and a somewhat higher number of reticulocytes, urine had a high concentration of delta aminolevulinic acid. The complex of symptoms indicates the initial stage of chronic intoxication with:

Explanation

Delta-aminolevulinic acid dehydratase is an enzyme necessary for the synthesis of heme. In lead poisoning, this enzyme is inhibited leading to anemia and an increased delta aminolevulinic acid content in urine. N/B a high reticulocyte number means an increase in RBC production ( new cells) and is observed in cases such as bleeding, high altitude or anemia. The above patient presents with signs of asthenic vegetative syndrome coupled with the above stated reasons confirming a case of lead poisoning.
122. Examination of a group of persons living on the same territory revealed the following common symptoms: dark-yellow pigmentation of the tooth enamel, diffuse osteoporosis of bone apparatus, ossification of ligaments and joints, functional disorders of the central. nervous system. This condition may be caused by the excessive concentration of the following microelement in food or drinking water:

Explanation

 

Presence of dark-yellow pigmentation of the tooth enamel, diffuse osteoporosis of bone apparatus, ossification of ligaments and joints are specific symptoms that indicate Fluorosis. This is a condition that occurs as a result of excessive intake of fluorine.

123. 45-year-old man complains of cough fits and tickling in his nasopharynx. He had been staying for 10 days in the polluted area created by the Chornobyl nuclear power plant accident. Rhinoscopy shows signs of severe nasopharynx irritation. What radionuclide is the cause of this irritation?

Explanation

 

Amongst the listed radioactive substances, an exposure to radioactive iodine  will result in the irritation of the nasopharynx, increased tear production, swelling of the neck, loss or a change in taste etc. Note that radioactive iodine is mostly used in the treatment of thyroid cancers and is readily absorbed by the thyroid gland. Majority of the other listed radioactive substances will lead to acute radiation sickness, skin burns etc.

124. A 28-year-old man complains of skin rash and itching on the both of his hands. The condition persists for 1,5 years. The exacerbation of his condition he ascribes to the occupational contact with formaldehyde resins. Objectively: lesion foci are symmetrically localized on both hands. Against the background of erythema with blurred margins there are papulae, vesicles, erosions, crusts, and scales. What is the most likely pathology?

Explanation

 

Eczema (dermatitis) is a chronic inflammatory disease of the skin characterized by the presence of red, itchy, dry scaly rashes. Occupational eczema (dermatitis) is gotten from contact with certain chemicals in the course of working (in this case due to contact with formaldehyde). Allergic dermatitis is a form of contact dermatitis that results from immune reactions towards certain irritants.

125. During assessment of work conditions at the mercury thermometer manufacture, content of mercury vapors in the air of working area is revealed to exceed maximum concentration limit. Specify the main way of mercury penetration into the body:

Explanation

 

Note that the question indicates “content of mercury vapors in the air of the working area”. Since the mercury is in vaporized form, the route of entry will be through the respiratory organs.

126. During health assessment of car drivers and police officers on point duty, the physicians detected carboxyhemoglobin in the blood of the patients, weakened reflex responses, disturbed activity of a number of enzymes. Revealed professional health disorders are most likely to be associated with the effect of

Explanation

Hemoglobin is responsible for the transport of oxygen in blood. It binds to oxygen molecules forming Oxyhemoglobin. In the presence of carbon monoxide, carbon attaches to hemoglobin (dissociates oxygen) forming carboxyhemoglobin thereby leading to a reduction in oxygen transport. In CO2 poisoning, Carbhemoglobin will be the complex formed.
127. On laboratory investigation of a pork sample there is 1 dead trichinella detected in 24 sections. This meat should be:

Explanation

The meat is infected by Trichinella and should be properly disposed.
128. Examination of a 43-year-old man objectively revealed pallor of skin and mucous membranes, loss of tongue papillae, transverse striation of fingernails, cracks in the mouth corners, tachycardia. Blood test results: Hb- 90 g/l, anisocytosis, poikilocytosis. The most likely causative factor of this condition is the inadequate intake of:

Explanation

 

The symptoms listed in the question stem are classic for Iron deficiency anemia.

A person with insufficient Iron in the blood will experience symptoms such as; fatigue, headaches, temperature sensitivity, heart palpitations, cold hands and feet, pallor of mucous membrane, cracks on the side of the mouth, inflammation of the tongue, brittle nails etc. These patients also experience an Iron deficiency anemia which is characterised by a reduction in hemoglobin level and color index.

The other microelements listed as answer choices are not directly involved with the development of anemia.

129. A 47-year-old man is employed at the weaving workshop, has 15-year-long record of service at this factory; his work conditions are associated with high frequency and high-intensity noise. During periodical examination he was diagnosed with occupational deafness. What are the grounds for making such a diagnosis?

Explanation

 

An audiometry focuses on measuring the hearing acuity (sharpness), check for variations in pitch, intensity, tones and frequencies. The above patient is diagnosed with occupational deafness (loss of hearing ability due to work hazards), to put up such diagnosis, an audiometry test is to be carried out coupled with assessing hygienic conditions of his workplace.

130. A regional cardiologist is tasked with the development of a plan for medioprophylactic measures aimed at decrease of cardiovascular mortality. What measures should be planned for secondary prevention?

Explanation

Primary prevention of a disease is aimed at preventing the onset (start of a disease) i.e., the disease is yet to occur so we aim at preventing it from happening via optimization of lifestyle and living conditions, vaccinations/Immunizations etc.

Secondary prevention is aimed at preventing or slowing the progress of a disease (early diagnosis and treatment) i.e., the patient already has the disease, so we are more concerned on stopping it from progressing and preventing its complications.

 
131. Clinic of a research institute for occupational diseases examined a worker who works at a concentration plant and diagnosed him with chronic dust bronchitis. The case is investigated by a commission including the representatives of: the plant, clinic, local SES, department of Social Insurance Fund, trade union. According to the ”regulation on investigation of. . . ”, the commission should be headed by the representative of the following authority:

Explanation

 

This committee should be headed by the local SES (state emergency committee) who are responsible for providing assistance especially in emergency situations to their immediate environment.

132. Bacterial analysis of air in a living space in winter period by means of Krotov’s apparatus revealed that total number of microorganisms in 1 m3 of air was 7200. What is the allowed number of microorganisms for the air to be characterized as ”pure”?

Explanation

Air cleanness estimation according to indexes of bacteriological research in different year periods.

 

Air estimation

Microorganisms Contents in 1m3 of air.

 

Generally

Streptococcus hemolyticus

Generally

Streptococcus hemolyticus

 

Summer period

Winter period

Clean

less 1500

less 16

less 4500

less 36

Moderately muddy

1500 - 2500

16-35

4500-7000

36-125

Muddy

more 2500

more 35

more 7000

more 125

133. In a pre-school educational establishment the menu consists of the following dishes: milk porridge from buckwheat, pasta with minced meat, cucumber salad, kissel (thin berry jelly), rye bread. What dish should be excluded from the menu?

Explanation

134. A 48-year-old woman has thermal burns of both hands. The epidermis of the palms and backs of her hands is exfoliating, and blisters filled with serous liquid are forming. The forearms are intact. What diagnosis is most likely?

Explanation

Rule of “nines” – area of different areas of the body is proportional: anterior surface of the trunk – 18 %, posterior – 18 %, lower limb – 18 %, external genitals – 1 %);

Rule of  “palm”. It is used if burns are limited and located on different areas of the body.

According to the rule of palm takes 1 % of the skin surface.

Division of the burns on superficial (I, II, IIIA st.) and deep (IIIB-IV st.)

I stage – hyperemia of the skin

II stage – separation of epidermis with formation of bullas

III A stage – necrosis of superficial layers of the skin with saving of bulbs hair, sweat glands and sebaceous glands.

IIIB stage – necrosis of all the derma

IV stage – necrosis of the skin and underlying tissues.

Using the information provided in the question stem:

Blisters (bullae) - Stage II (blisters filled with serous liquid are forming)

Necrosis of superficial layers - Stage IIIA (epidermis of the palms and backs of her hands is exfoliating)

135. An electro-gas welding operator working at a machine workshop performs welding and cutting of metal, which is accompanied by intense UV-radiation. His welding station is equipped with efficient mechanical ventilation. What occupational disease is most likely to develop in an electro gas welding operator?

Explanation

 

The main risk amongst welders is the inflammation of the cornea and conjunctiva, commonly known as 'arc eye' or 'flash'. This is caused by the irritation of the eyes by the ultraviolet rays produced. Other possible hazards include skin inflammation, cancer.

136. Monthly dysentery morbidity in the regiongiven in absolute figures is as follows: January- 6; February - 9; March - 11; April - 10; May -16; June - 23; July - 19; August - 33; September - 58; October - 19; November - 11; December - 5. Annual total is 220 cases. What graphic presentation would provide the best visual for monthly deviations of dysentery morbidity from the average?

Explanation

Radar charts are also known as spider, polar, web charts or star plots; they are used as a way to visualize multivariate data. They are used to plot one or more groups of values over multiple common variables. They do this by giving an axis for each variable, and these axes are arranged radially around a central point and spaced equally.

A cartogram uses a map to differentiate how a variable is distributed.

 
137. A 50-year-old man, who works as a polisher at a combine building factory, addressed the factory’s sectorial doctor with complaints of general fatigue, sensations of numbness and pain in his fingers. Objectively: the skin of his fingers is pale. Reaction to pain, tactile and thermal stimuli was revealed to be slightly disrupted. No disruptions can be observed within the other organs and systems. What disorder is most likely?

Explanation

Pneumatic hammer disease is also known as  white fingers, traumatic vasospastic syndrome, vasospastic disease of the hands or hypothenar hammer syndrome, It occurs in individuals whose occupation requires the use of huge percussion machines such as hammers, drilling equipment  and also in sport related activities such as weightlifting, volleyball. This disorder is as a result disturbed blood flow to the affected hand.

Raynaud phenomenon possess similar physical changes but  usually occurs due to temperature changes or emotional distortions.

 
138. A worker, who was involved in fire fighting inside the building that stored 2 kg of mercury, has been delivered to a hospital with complaints of emotional expansiveness, palpitations, excessive sweating, body tremor, heart pain. Within one day his condition aggravated. Objectively: the skin is pale and moist. The patient is depressed. Permanent red dermographism, erethism, unstable BP are observed. What drug is the serum in this case?

Explanation

From anamnesis, we can tell this patient has had prior exposure to contents of mercury, this can  be confirmed by the presented symptoms which indicate poisoning by mercury. For the management of mercury poisoning, Unithiol should be administered; it is a chelating agent that is used as a remedy for poisonings by heavy metals. Other chelating agent include  dimercaprol, d-penicillamine etc.

Atropine sulphate is used as a remedy for organophosphate poisoning, calcium tetacine is used in lead poisoning, amyl nitrite is used for cyanide poisoning.

 
139. During meat testing Trichinella was detected in diaphragm crura in one of the two muscular tissue samples. What tactics should a doctor choose regarding this meat?

Explanation

 

This meat has been found to be infested by trichinella; if consumed, this individual is likely to get Trichinellosis. In this case, preservation of the meat by salting, boiling or freezing isn't going to salvage the meat. This meat should be properly disposed of to avoid consumption.

140. The following indicators were calculated to analyse population health and treatment quality in a cardiological hospital: primary cardiovascular morbidity - 62%; total cardiovascular morbidity - 483,55%; cardiovascular mortality - 10,9%; proportion of cardiovascular mortality within total mortality -67,0%; primary disablement caused by cardiovascular diseases - 16,2 per 10.000 population. What indicator is an extensive value?

Explanation

The extensive statistical indicator is used to determine a structure of a disease ( in this case, how many percent ( the proportion) of the total diseased fall under cardiovascular cases) ie., it shows, what part from the general number of all diseases is made with this or that disease which enters into total.

 

The intensive parameter characterizes frequency or distribution. It shows how frequently the given phenomenon occurs in the given environment.

141. After a lengthy march an army regiment has set camp for 3 days near a settlement. Sanitary hygienic investigation detected several water sources. Choose the source that would satisfy the demands for potable water the most under the given field conditions:

Explanation

 

Artesian well is a type of well from which water flows under natural pressure without pumping. It is dug or drilled wherever a gently dipping, permeable rock layer (such as sandstone) receives water along its outcrop at a level higher than the level of the surface of the ground at the well site. From the above listed options, the artesian well is the best source of clean water.

142. Clinical statistical investigation was performed to determine efficiency of a new pharmacological preparation for patients with ischemic heart disease. What parametric test (coefficient) can be used to estimate reliability of the results?

Explanation

The T-test is a statistical, hypothetical test used to compare the mean (average) of two related variables. For example comparing the mean of the experimental group to that of the controlled group. It lets you know how significant the differences are.

 

The Z-test also compares the means of two populations although it relies on the Variance ( the square root of the mean deviation).

143. A planner designs a heating system for a pre-school educational establishment. The highest air temperature should be in the following room:

Explanation

144. An emergency situation at a chemical plant caused acute occupational intoxication. A doctor who revealed the case of ”acute occupational disease (intoxication)” must notify the following authority:

Explanation

145. The process of open-cut mining requires drilling and blasting operations, rock and ore excavation, transportation of ore to fragmentation and sorting factories and transportation of barren rock to slag-heaps, road building and maintenance, repair works. What factor of production is most important for miner’s health?

Explanation

 

Mining involves the extraction of materials/ minerals from the earth; just like the question states, it involves processes such as drilling, blasting, excavation, sorting etc. Miners are usually exposed to lots of hazards such as earth collapse, dust inhalation, noise, vibrations etc. Of all the above listed hazards, exposure to dust is the most important/frequent hazard to a miners health; depending on the type of dust, this can lead to restrictive lung diseases such as pneumoconiosis, silicosis etc.

146. The correlation between the service record and eosinophil concentration in blood was studied in workers at dyeing shops of textile factories. What index will be most informative for the analysis of this data?

Explanation

 

Correlation factor/Coefficient is a statistical measure that shows the relationship between two variables; in the case between service records and eosinophil concentration. Most common types include the Pearson’s correlation coefficient and rank coefficient.

147. A region attended by a central regional hospital demonstrates increased hemorrhagic stroke morbidity. Essential hypertension morbidity, however, remains at the same level and is below the average level registered within the larger area. What managerial decision should be made in this case?

Explanation

148. Estimation of community health level involved analysis of a report on diseases registered among the population of district under charge (reporting form 12). What index is calculated based on this report?

Explanation

 

Common morbidity rate refers to the proportion or frequency at which a disease occurs in a population; from the report of registered diseases in a particular area, we can calculate the  morbidity rate.

149. A 45-year-old patient (14-year-long work record as a house painter) upon the contact with synthetic paint develops skin reddening, edema, severe itching and oozing lesions on her face. Symptoms disappear after the contact with this chemical substance stops but even the smell of paint alone is enough to make them reappear each time. Each recurrence is characterised by increased severity of symptoms. What provisional diagnosis can be made?

Explanation

Eczema (dermatitis ) is a chronic inflammatory disease of the skin characterized by the presence of red, itchy, dry scaly rashes. Occupational or professional  eczema (dermatitis) is gotten from contact with certain chemicals in the course of working (in this case due to contact with the chemicals in paint ). Allergic dermatitis is a form of contact dermatitis that results from immune reactions towards certain irritants.
150. A 38-year-old man’s workplace is within the area of effect of ionizing radiation. During regular medical check up he expresses no complaints. Blood test: erythrocytes -4, 5 · 1012/l, Нb- 80 g/l, leukocytes - 2, 8 · 109/l, platelets - 30 · 109/l. Can this person continue to work with sources of ionizing radiation?

Explanation

 

Observe that the levels of WBC (Leukocytes), hemoglobin and platelets are very low. This can be traced to the effect of radiation from around his workplace. Working in this environment should be prohibited for this individual because of the health effects.

Complete Blood Count
151. During medical examination of a group of children under 4 years carried out by a pediatric team in one of the African countries a set of similar pathological signs was detected in some of the children. The signs are as follows: growth inhibition, mental changes, muscle atrophy, swellings, changes in hair and skin pigmentation. These children were diagnosed with kwashiorkor. What food products should be added to the diet to treat this disorder?

Explanation

 

Kwashiorkor is a disease marked by severe protein malnutrition and bilateral extremity swelling. It usually affects infants and children. It should be differentiated from marasmus which is energy or calorie deficiency. For the correction of kwashiorkor, foods rich in protein should be taken eg, fish, meat, cereals, milk etc.

152. A patient with high temperature came to a first aid post in the evening. The fact of temporary disability was established. Indicate the measure to be taken in this case:

Explanation

 

The physician on duty should issue a medical certificate ( a written statement from a physician or another medically qualified health care provider which attests to the result of a medical examination of a patient) indicating a temporary disability. This medical certificate will assist the patient in obtaining a sick list which is a document given to an individual exempting him/her from workday to an illness or disability.

153. Clinic of a research institute for occupational diseases examined a worker who works at a concentration plant and diagnosed him with chronic dust bronchitis. The case is investigated by a commission including the representatives of: the plant, medical unit, territorial sanitation center, department of Social Insurance Fund, trade union. According to the \"regulation on investigation of. . . \", the commission should be headed by the representative of the following authority:

Explanation

154. Maximum permissible concentration of carbon dioxide in the air is considered to be a sanitary index of air purity in a classroom. What concentration of carbon dioxide in the air is accepted as a permissible maximum?

Explanation

CO2 maximum allowable concentration (MAC) for indoors is determined at the level 0.07%-0.1%. In industrial premises where CO2 is accumulated during manufacture processes- 1-1.5%. The indoor air is considered pure if the CO2 concentration does not exceed the maximum allowable concentration 0.07% by Pettencofer or 0.1% by Flugge.
155. A patient with cardiac infarction is on sick leave with his medical certificate being continuously extended for 4 months according to a standard procedure. The disease is progressing. The issue is put forward to refer the patient to the industrial injury assessment board. Who among the medical staff of medical and preventive treatment facility is authorized to do this?

Explanation

The right of prolongating a medical certificate of invalidity alongside with the medical treatment by the doctor is granted by:

• Chief medical officer (assistant of the chief medical officer on expertise of ability for work);

• Manager of department;

• Medical-consulting commission also known as medical expertise committee

The medical-consulting commission (MCC) is formed in case the doctors staff is above 15 doctors.

 
156. A 46-year-old patient once took part in elimination of breakdown at an atomic power plant. Currently he is being treated at an inpatient hospital unit. He was diagnosed with progressing somatoform autonomic dysfunction. This disease relates to the following group of ionizing radiation effects:

Explanation

 

Biological Effects of radiation are of two types a) 1.Deterministic Effect b) Stochastic Effect. Deterministic effects are also called non-stochastic effects. These effects depend on time of exposure, doses, type of Radiation.it has a threshold of doses below which the effect does not occur, the threshold may vary from person to person. They include acute and chronic graduation sickness. Stochastic effects are those effects which occur when a person receives a high dose of radiation. These effects have an increased probability of occurring with increased dose. The two types include a) somatic stochastic  effect eg., somatoform autonomic dysfunction, malignancies etc  B) Genetic or hereditary effects.

157. Over a current year among workers of an institution 10% have not been ill a single time, 30% have been ill once, 15% - twice, 5% - 4 times, the rest - 5 and more times. What is the percentage of workers belonging to the I health group?

Explanation

158. In river-side urban community there was an outbreak of hepatitis type A possibly spread by water. What indexes of river water quality can confirm this theory?

Explanation

 

Coliphages are microbial indicators specified in the Ground Water Rule (GWR) that can be used to monitor for potential fecal contamination of drinking water. Number of bacteria of group of an intestinal rod (coli-index) in 1000 ml of water - no more than 3.Common microbial number in one ml of water - no more than 100.

159. Establishments participating in medical examinations include: medical and preventive treatment facility, hygiene and preventive treatment facility, sociomedical expert committees, Ministry of Defence medical committees, Ministry of Domestic Affairs medical committees, forensic medicine agency, etc. Specify what service deals with sociomedical assessment of temporary disability:

Explanation

160. In order to study impact of microclimate on the human organism it is necessary to make systematic observation of air temperature during the period of 3 days. Choose a device that will allow to make the most precise temperature records:

Explanation

 

A thermograph is a visual representation of temperature recordings of an area over a period of time. Thermometers are used to measure instant temperature while a psychrometer is used in measuring relative humidity.

161. Various population groups mortality has been studied for a long time; territorial distribution of population and resulting changes in mortality trends are considered. What statistical method can be applied?

Explanation

162. A workshop resident doctor makes a list of workers, who are often ill, for special supervision. He takes into account the number of etiologically connected cases resulting in temporary disability that occurred in the span of a year for each worker. How many such cases should a worker have to be included in this group?

Explanation

163. A tractor driver with the record of service of 24 years has undergone palestesiometry test (Vibration Sensitivity Measurement). Test revealed increased vibration sensitivity threshold at the frequencies of 63-125-259Hz to 25 dB. Dynamometry is 20 kg on the right and 16 kg on the left. Cold stimulus test is positive, time of hand temperature restoration is 52 minutes. Blanching at pressure symptom is positive and equals 21. Hypesthesia of upper and lower limbs is observed and can be classified as \"gloves\" and \"socks\" polyneuritic pattern. Make the provisional diagnosis.

Explanation

According to the character of influence to the organism there is local, general and combined vibration. During the local vibration the transmission of mechanical oscillations to the body is realized through the arms. Most often this etiologic factor we can see among the face-workers, chasers, drillers. The sources of the general vibration are the vibro platform, the table vibrator, the forming and concrete-laying machines.

The combined vibration is a combination of local and general - as in this case with a tractor driver who will have both localized and generalized vibratory impact.

According to the intensity’s degree of pathologic process symbolically mark out 4 stages of disease:

I – initial (episodic finger numbness with or without tingling - Hypesthesia)

II – moderately expressed (dystrophic disorders);

III – expressed (irreversible organic changes);

 

IV – generalized

164. Head of a department and a trade union group have appealed to the head of a hospital about dismissal of the senior nurse who has 17 year record of service. The facts of charge were confirmed and recognized by the nurse herself. This nurse lives with a daughter (who is divorced and unemployed) and a 9-month-old grandson. Make an administrative decision:

Explanation

165. Deputy of chief medical officer carried out a study of morbidity rate for population which had been served at the polyclinics within the last 5 years. What statistical values can help in calculation of morbidity rates?

Explanation

166. A 50-year-old locksmith has a long term record of work under the effect of mercury vapors with concentration exceeding MPC by 5-10 times. Clinical examination revealed the lability of vasomotors of skin, pulse and arterial pressure; total hyperhydrosis; asymmetric innervation of facial and lingual muscles, positive subcortical reflexes, intention tremor. Against the background of increased emotional excitability the patient presents with lack of self-confidence, shyness. A dentist found him to have parodontosis, chronic stomatitis. What disease can be suspected?

Explanation

167. A 36-year-old female patient complains of bruises on the body, gingival haemorrhage, general weakness. A month ago she had a severe domestic poisoning with some pesticide (the patient can not remember the name). She has a 7-year record of working in contact with petroleum products, particularly benzene. In blood: RBCs - 3, 2 · 1012/l, WBCs -2, 7 · 109/l, thrombocytes - 70 · 109/l. What is the most likely pathology?

Explanation

168. Educational rooms are illuminated with various lighting fittings. What type of lighting fittings is the most appropriate in respect of hygienic norms?

Explanation